Trapazoidal Approximation help

  • Thread starter Thread starter Asphyxiated
  • Start date Start date
  • Tags Tags
    Approximation
Click For Summary
The discussion focuses on calculating the definite integral of the function f(x) = x/(x+1) from 1 to 5 using the trapezoidal approximation with dx = 0.5. The initial calculations yield a result of approximately 2.921789, while the textbook answer is 2.8968. Errors were identified in the summation of the trapezoidal areas, specifically with the value for x4, and in the calculation of the error term, where dx was mistakenly noted as 0.25 instead of 0.5. After correcting these mistakes, the calculations aligned with the expected results. The conversation highlights the importance of careful arithmetic and attention to detail in numerical approximations.
Asphyxiated
Messages
263
Reaction score
0

Homework Statement



\int^{5}_{1} \frac{x}{x+1} dx

using dx = .5

Homework Equations



\sum^{a}_{b} \frac {f(x)+f(x+dx)}{2} dx = [\frac{1}{2}f(x_{0})+f(x_{1})+\cdots+f(x_{n-1})+\frac{1}{2}f(x_{n})]dx

The Attempt at a Solution


TAI = Trapazoidal Approximation Input value

x_{0}=1 \rightarrow f(x_{0})=\frac {1}{2} \rightarrow TAI = \frac{1}{2}\frac{1}{2} =\frac{1}{4}

x_{1} = 1.5 \rightarrow f(x_{1}) = \frac{1.5}{2.5} \rightarrow TAI = \frac{1.5}{2.5}

x_{2} = 2 \rightarrow f(x_{2}) = \frac{2}{3} \rightarrow TAI = \frac{2}{3}

x_{3} = 2.5 \rightarrow f(x_{3})= \frac{2.5}{3.5} \rightarrow TAI = \frac {2.5}{3.5}

x_{4} = 3 \rightarrow f(x_{4}) = \frac{4}{5} \rightarrow TAI = \frac {4}{5}

x_{5} = 3.5 \rightarrow f(x_{5})= \frac{3.5}{4.5} \rightarrow TAI = \frac {3.5}{4.5}

x_{6} = 4 \rightarrow f(x_{6}) = \frac {4}{5} \rightarrow TAI = \frac {4}{5}

x_{7} = 4.5 \rightarrow f(x_{7}) = \frac{4.5}{5.5} \rightarrow TAI = \frac {4.5}{5.5}

x_{8} = 5 \rightarrow f(x_{8}) = \frac {5}{6} \rightarrow TAI = \frac{1}{2}\frac{5}{6} = \frac {5}{12}

so now, following the formula I add up all the TAI values, and the 1/2's have already been applied, so:

\frac{1}{4} + \frac {1.5}{2.5} + \frac{2}{3} + \frac{2.5}{3.5} + \frac {3}{4}+ \frac{3.5}{4.5} + \frac {4}{5} + \frac {4.5}{5.5} + \frac {5}{12} = 5.843578644 = \frac{20248}{3465}

and then I multiply the sum by dx which is .5:

\frac {20248}{3465} * \frac {1}{2} = 2.921789

the book states that the answer they are looking for is 2.8968

and for E(dx) (error) I take the second derivative:

y = \frac {x}{x+1}

y' = \frac {(x+1)(1)-(x)(1)}{(x+1)^{2}}

y' = \frac {x+1-x}{(x+1)^{2}}

y' = \frac {1}{(x+1)^{2}}

which the same as:

y' = (x+1)^{-2}

so:

y''= -2(x+1)^{-3}

and the formula for error is:

|f''(x)| \leq M for a \leq x \leq b

and:

E(dx)= \frac {b-a}{12} M (dx)^{2}

so now:

|f''(1)| = .25

|f''(5)| = .00926

so f'' has M at 1 so to find error we do:

\frac {5-1}{12}(.25)(.25)^{2} = \frac {1}{192}

the book states that the correct E(dx) is \frac {1}{48}

I have done this many times over and can not find where I went wrong, especially with the actually summing in the first part.
 
Physics news on Phys.org
For x4 you put down 4/5 instead of 3/4. You wrote the right thing in the horizontal addition, but the sum you come up with is the sum including 4/5 instead of 3/4.

In the error part, your dx is 0.5, not 0.25.
 
hmm i just mixed up on the post with the 4/5 instead of 3/4, that's what I have written on the paper but i just did the problem again and it all works out, i wonder why i couldn't get it to come out before... and the error, yeah i got mixed up, thanks for the help!
 
Question: A clock's minute hand has length 4 and its hour hand has length 3. What is the distance between the tips at the moment when it is increasing most rapidly?(Putnam Exam Question) Answer: Making assumption that both the hands moves at constant angular velocities, the answer is ## \sqrt{7} .## But don't you think this assumption is somewhat doubtful and wrong?

Similar threads

Replies
2
Views
2K
Replies
6
Views
2K
Replies
4
Views
2K
  • · Replies 105 ·
4
Replies
105
Views
6K
Replies
17
Views
2K
Replies
2
Views
2K
  • · Replies 7 ·
Replies
7
Views
2K
Replies
4
Views
2K
  • · Replies 2 ·
Replies
2
Views
2K
  • · Replies 4 ·
Replies
4
Views
2K